De digitale vraagbaak voor het wiskundeonderwijs

home |  vandaag |  gisteren |  bijzonder |  gastenboek |  wie is wie? |  verhalen |  contact

HOME

samengevat
vragen bekijken
een vraag stellen
hulpjes
zoeken
FAQ
links
twitter
boeken
help

inloggen

colofon

  \require{AMSmath}

Reageren...

Re: Re: Functie bepalen na vermenigvuldiging vlakdeel

Hallo,

Ik loop bij een opgave over het berekenen van de maximale lengte vast. Ten eerste is het mij eigenlijk niet echt duidelijk, wat ze willen dat ik ga berekenen? Willen ze het stuk A tot punt Z (fictief), Z staat dan loodrecht onder P?



Verder zag ik bij de uitwerkingen dat ze da/dx gebruiken, maar dat is mij eigenlijk niet helemaal duidelijk waarom ze een afgeleide gebruiken? Ik dacht dat je dit met een meetkundige vergelijking kon oplossen, maar ze berekenen de afgeleide naar 0. En hiermee berekenen ze x.



En uiteindelijk nemen ze voor het eindantwoord de x + (√5 · -x - (x)2). En die laatste regel snapte ik helemaal niet. Ik hoop dat iemand mij hierbij kan helpen?

Mvg,
Anna

Antwoord



In driehoek ARB is AZ gelijk aan $a$. Die $a$ is de lengte van het stuk van de garagedeur dat uitsteekt. In deze rechthoekige driehoek geldt:

$AR^2+BR^2=AB^2$

Invullen geeft:

$(a+x)^2+(2,5-x)^2=2,5^2$

Je kunt nu $a$ uitdrukken in $x$. Oplossen naar $a$ (zie uitwerkingenboek) geeft:

$
a = - x + \sqrt {5x - x^2 }
$

Je hebt nu $a$ geschreven als functie van $x$. Je wilt weten voor welke waarde van $x$ die $a$ maximaal is. Dat is dan een kwestie van de afgeleide bepalen, de afgeleide nul stellen, grafiek plotten en vaststellen of er sprake is van een maximum of minimum. Een optimaliseringsprobleem dus...

De afgeleide wordt:

$
\eqalign{a' = - 1 + \frac{{5 - 2x}}
{{\sqrt {5x - x^2 } }}}
$

Op nul stellen en oplossen geeft als mogelijke kandidaat voor een extreem:

$
\eqalign{x = \frac{5}
{2} - \frac{{5\sqrt 2 }}
{4}}
$

Als je (met een schets/plot/tekenverloop) hebt vastgesteld dat het maximum bij deze $x$-waarde wordt aangenomen vul je de $x$-waarde in bij de functie. Dat geeft je dan de maximale waarde voor $a$:

$\eqalign{a=-\left({\frac{5}{2}-\frac{{5\sqrt2}}{4}}\right)+\sqrt{5\left({\frac{5}{2}-\frac{{5\sqrt2}}{4}}\right)-\left({\frac{5}{2}-\frac{{5\sqrt2}}{4}}\right)^2}\approx{\text{1}}{\text{,04}}}$

Opgelost!

Je moet maar even aangeven waar en/of wat er nog iets onduidelijk is. Hopelijk helpt dat.

Gebruik dit formulier alleen om te reageren op de inhoud van de vraag en/of het antwoord hierboven. Voor het stellen van nieuwe vragen kan je gebruik maken van een vraag stellen in het menu aan de linker kant. Alvast bedankt!

Reactie:

Klik eerst in het tekstvlak voordat je deze knopjes en tekens gebruikt.
Pas op: onderstaande knopjes en speciale karakters werken niet bij ALLE browsers!


áâæàåãäßçéêèëíîìïñóôòøõöúûùüýÿ½¼¾£®©




$\mathbf{N}$ $\mathbf{Z}$ $\mathbf{Q}$ $\mathbf{R}$ $\mathbf{C}$
Categorie: Integreren
Ik ben:
Naam:
Emailadres:
Datum:18-5-2024